120 Unit 2- Omness

Pataasin ang iyong marka sa homework at exams ngayon gamit ang Quizwiz!

Tell the mother that acetaminophen and rest should help to alleviate these symptoms, which are not unusual after immunizations. These minor reactions may be treated with acetaminophen and rest. Reactions that are more serious or unusual should be reported with a VAERS form.

After immunizations at a well-baby check-up, a mother notes that her infant has redness and swelling at the injection site, is irritable, and has a fever of 99.9° F. She calls the clinic and is worried about her baby. Which action by the nurse is appropriate? Tell the mother there's nothing to worry about and that these symptoms are normal and will go away. Tell the mother that she should bring her baby in to the office for an examination. Tell the mother that acetaminophen and rest should help to alleviate these symptoms, which are not unusual after immunizations. Complete a VAERS form.

Has little control over this behavior Emotional instability usually is caused by lesions affecting the thalamic area (the part of the neural system most responsible for emotions). Crying easily is not attention-getting behavior; lability of mood is a physiological response to the CVA. The client may have remote memory, but there is no selective process of what events are remembered. There are inadequate data to come to the conclusion that the client feels guilt. Lability of mood is a physiological response to the CVA.

Family members of a client who had a brain attack (cerebrovascular accident, CVA) ask why the client cries easily and without provocation. How does the nurse explain the client's behavior? Is making an attempt to get attention Has selective memory from the past, especially the sad events Has little control over this behavior Feels guilty about the demands being made on the family

Neural tube defect Increased levels of alpha-fetoprotein in pregnant women have been found to reflect open neural tube defects such as spina bifida and anencephaly. Cystic fibrosis is a genetic defect that is not associated with the AFP level. A Guthrie test soon after ingestion of formula can determine whether an infant has phenylketonuria. Down syndrome is a chromosomal defect that is associated with a low AFP level.

A pregnant client's blood test reveals an increased alpha-fetoprotein (AFP) level. Which condition does the nurse suspect that this result indicates? Cystic fibrosis Phenylketonuria Down syndrome Neural tube defect

Immunoglobulins Vaccines, antitoxins, and toxoids provide active immunity by stimulating the humoral immune system. Immunoglobulins provide passive immunity by giving the patient substances to fight specific antigens.

Administration of which substance provides passive immunity? Vaccines Antitoxins Immunoglobulins Toxoids

Antiinflammatory The antiinflammatory action of aspirin reduces joint inflammation. Aspirin reduces fever, but this is not the rationale for prescribing it for clients with rheumatoid arthritis. Aspirin does not preserve bone integrity. Flexion contractures are prevented by exercise, not aspirin.

Aspirin is prescribed on a regular schedule for a client with rheumatoid arthritis. The nurse understands that the drug is being used primarily for which of its properties? Analgesic Antipyretic Antiinflammatory Antiplatelet

toxoids

Bacterial exotoxins that are weakened or inactivated (by chemicals or heat) so that they are no longer toxic but can stimulate the immune system to produce an antibody; a type of vaccine and therefore a type of active immunizing drug.

yeast. The nurse should not administer the papillomavirus vaccine (Gardasil, Cervarix) to patients with allergies to yeast or patients who have a documented allergic reaction to the first injection of the vaccine.

Before administering Gardasil, it is most important for the nurse to assess the patient for an allergy to: yeast. milk. eggs. penicillin.

sexual acts

Behaviors that include genitalia and erogenous zones

passive immunizing drugs

Drugs containing antibodies or antitoxins that can kill or inactivate pathogens by binding to the associated antigens. These are directly injected into a person and provide that person with the means to fend off infection, bypassing the person's own immune system

hyperaemia

Increased blood flow to a part of the body

antibody titer

Measure of how many antibodies to a given antigen are present in the blood and is used to assess whether enough antibodies are present to protect the body effectively against the particular pathogen

sex

One of four primary drives (thirst, hunger, avoidance of pain

neurotransmission

Process of sending signals from nerve to nerve across a synapse

reticuloendothelial system

Specialized cells located in the liver, spleen, lymphatics, and bone marrow that remove miscellaneous particles from the circulation, such as aging antibody molecules

biologic antimicrobial drugs (biologics)

Substances of biologic origin used to prevent, treat, or cure infectious diseases (e.g., antitoxins, vaccines, toxoids, immunoglobins).

active immunizing drugs

Toxoids or vaccines that are administered to a host to stimulate host production of antibodies

Artificial active immunization

Type of immunization in which the body is clinically exposed to a relatively harmless form of an antigen that does not cause an actual infection

Anaphylaxis Anaphylaxis is a potential life-threatening adverse reaction to vaccines. Pain and myalgias can occur but are not life threatening.

What is the priority nursing assessment to monitor when administering vaccinations? Anaphylaxis Myalgias Pain at the injection site Symptoms of infection

sexual identity

Whether one is male or female based on biological sexual characteristics

stereotactic biopsy

allows precise localization of a specific area of the brain

↑ICP

caused by a pathological condition or trauma that causes pressure within the cranial vault to increase

sedatives

class of medication that decreases ICP by reducing metabolic rate; propofol is a commonly used agent

immunoglobulins (Ig)

glycoproteins synthesized and used by the humoral immune system (B cells) to attack and kill all substances foreign to the body; a type of passive immunizing drug

acute inflammation

immediate inflammatory response to tissue injury that is short in duration (minutes to days)

≥20mm Hg

intercranial hypertension (value)

≤15m Hg

optimal ICP

intra

prefix that refers to within

gestation

the process of carrying or being carried in the womb between conception and birth

Adenoids Appendix Bone marrow The lymphoid organs of the immune system are the adenoids, appendix, and bone marrow. Other organs of the immune system include the lymph nodes, thymus gland, tonsils, and spleen. The gallbladder, liver, and thyroid gland are not part of the immune system.

Select all the organs from the following list that are part of the immune system. Select all that apply. Adenoids Appendix Bone marrow Gallbladder Liver Thyroid gland

4 to 8 weeks. The embryonic period lasts from the beginning of the fourth week to the end of the eighth week. Teratogenicity is a major concern because all external and internal structures are developing in the embryonic period. A woman should avoid exposure to all potential toxins during pregnancy, especially alcohol, tobacco, radiation, and infectious agents. At the end of this period, the embryo has human features. The span of gestation from 8 to 12 weeks, from 12 to 16 weeks, or from 16 to 20 weeks is not within the embryonic stage of fetal development, when teratogenicity is of greatest concern.

The embryonic period is critical because external and internal structures in the fetus are forming. All teratogens should be avoided from 4 to 8 weeks. 8 to 12 weeks. 12 to 16 weeks. 16 to 20 weeks.

Vaccine Adverse Event Reporting System (VAERS)

a United States program for vaccine safety, co-managed by the CDC and FDA; collects information about adverse events that occur after administration of vaccines.

contraception

a method or device used to prevent pregnancy

tetanus immunoglobulin (BayTet)

a passive immunizing drug effective against tetanus. given to persons with tetanus-prone wounds, or to treat active tetanus.

Reye's syndrome

a serious condition that causes confusion, swelling in the brain, and liver damage; can result from aspirin administration in children

anaphylaxis

a severe, potentially life-threatening allergic reaction consisting of rash, difficulty breathing, and shock; a severe adverse reaction to vaccine administration

craniotomy

a surgical operation in which a bone flap is temporarily removed from the skull to access the brain; may be performed to remove a lesion or tumor, repair a damages area, relieve pressure, or drain blood

IUD (intrauterine device)

a type of contraceptive that prevents implantation

oral contraceptive pill

a type of contraceptive that prevents ovulation; 99% effective

Stop the transfusion immediately. The client is experiencing an allergic reaction to the transfusion. The nurse should stop the transfusion immediately. The health care provider then should be notified. Flushing red blood cells with dextrose and normal saline will cause hemolysis and will not be effective in stopping the reaction. Slowing down the rate will make the situation worse.

The primary healthcare provider prescribes one unit of packed red blood cells to be administered to the client who suffered a hip fracture. Several minutes after the start of the infusion, the client reports itching. Upon further assessment, the nurse observes hives on the client's chest. Which action should the nurse take next? Call the primary healthcare provider to obtain a prescription for an antihistamine. Flush packed red blood cells with 5% dextrose and 0.45% normal saline. Slow down the rate of the infusion. Stop the transfusion immediately.

Glascow Coma Scale (GCS)

aims to give a reliable and objective way of recording the state of a person's consciousness for initial as well as subsequent assessment. A person is assessed against their motor response to verbal commands, eye opening in response to speech, and verbal response to speech, and the resulting points give a person's score between 3 (indicating deep unconsciousness) and 15.

secondary immunodeficiency

an acquired suppressed immune response either intentionally, or as an adverse effect of treatment

inflammation

an immunologic defense against tissue injury, infection, or allergy

Acetaminophen Rest Warm compress

recommendations for patients showing minor adverse effects of vaccine administration (3)

antigens

substances, usually proteins and foreign to a host, that stimulate the production of antibodies and that react specifically with those antibodies; includes bacterial exotoxins, viruses, and allergens.

vaccines

suspensions of live, attenuated, or killed microorganisms that prevents the injected person from contracting the disease. They are another type of active immunizing drug

varicella-zoster immunoglobulin (VZIG)

passive immunity used to modify or prevent chickenpox in susceptible individuals who have has recent significant exposure to the disease. given to those at high risk for serious complications if they become infected with the virus, including newborn children, and immunocompromised adults.

-partners -practices -protection -past history -pregnancy prevention

the 5 P's of sexual history taking

conception

the beginning of pregnancy involving fertilization and implantation of the embryo onto the uterine wall

cranium

the collective bone structure that encloses the brain, also known as the skull

brain tissue (80%) blood (10%) cerebrospinal fluid (CSF) (10%)

the components of the skull (3)

intracranial

the components that lie within the skull, including the brain, circulatory system, and dura mater

innate immunity

the immunity present at birth

Apply warm compresses to the injection site. Applying warm compresses to the injection site and using acetaminophen (not aspirin, which carries the risk for Reye's syndrome) should help to relieve the discomfort. The other options are incorrect.

A 12-month-old infant has received an MMR II (measles, mumps, and rubella virus vaccine), and her mother calls the clinic that afternoon to ask about helping her fussy infant to "feel better." What will the nurse suggest? Apply an ice pack to the injection site. Apply warm compresses to the injection site. Observe the site for further swelling and redness. Bring the infant in to the emergency department for an immediate examination.

Physical therapy Rehabilitation needs for a client with Guillain-Barré syndrome focus on physical therapy and exercise for the lower extremities because of muscle weakness and discomfort. A client with Guillain-Barré syndrome does not need speech or swallowing exercises. A client with Guillain-Barré syndrome does not need vertebral support. Problems with cataracts are not associated with Guillain-Barré syndrome.

A nurse is caring for a client with Guillain-Barré syndrome. The nurse should prepare the client for what essential care related to rehabilitation? Physical therapy Speech exercises Fitting with a vertebral brace Follow-up on cataract progression

"Enteric coated medications are absorbed lower in the digestive tract and can be irritating to my intestines or inadequately absorbed by my inflamed tissue." Enteric coatings on medications are designed to prevent breakdown and absorption of the medication until lower in the digestive tract, usually to prevent stomach irritation or to reach a certain point in the digestive tract for optimal absorption. For the patient with ulcerative colitis, the intestinal lining is inflamed or susceptible to inflammation and can have impaired absorption; therefore, enteric coated medications should be avoided. The coating is not irritating, but the medication can be. The response time of the medication is not a concern in this instance. Enteric coated medicines do not cause diarrhea simply because they are enteric coated.

A nurse is instructing her patient with ulcerative colitis regarding the need to avoid enteric coated medications. The nurse knows that the patient understands the reason for this teaching when he states which of the following? "The coating on these medications is irritating to my intestines." "I need a more immediate response from my medications than can be obtained from enteric coated medications." "Enteric coated medications are absorbed lower in the digestive tract and can be irritating to my intestines or inadequately absorbed by my inflamed tissue." "I don't need to use these medications because they cause diarrhea, and I have had enough trouble with diarrhea and rectal bleeding over the past weeks."

amniocentesis

medical procedure used in prenatal diagnosis of chromosomal abnormalities and fetal infections, and also for sex determination, in which a small amount of amniotic fluid is sampled from the amniotic sac surrounding a developing fetus

It is given by subcutaneous injection. Pneumococcal vaccines are given by subcutaneous injection, and only once. The other options are incorrect.

A 72-year-old patient is to receive a pneumococcal vaccine, polyvalent (Pneumovax 23). The nurse recognizes that which statement about this vaccine is true? It is given by subcutaneous injection. It is given by deep intramuscular injection. The patient will have to return for a total of three injections. It is given yearly to provide immunization against current strains of pneumococcal bacteria.

neurotransmitter

Chemical that transmits signals from a neuron to a target cell

Impotence due to alcohol ingestion Factors affecting male infertility include impotence due to alcohol. Nightly masturbation and high testosterone levels do not have the greatest implication on male infertility in a patient with admitted alcohol issues. Primary anovulation refers to female infertility, so it is not a consideration for this question about male infertility.

The RN at the Preconception Counseling Clinic takes a male history for infertility evaluation. Which finding has the greatest implication for this patient's care? Practice of nightly masturbation Primary anovulation High testosterone levels Impotence due to alcohol ingestion

11 years of age and older Currently, DTaP is the preferred preparation for primary and booster immunization against these diseases in children from 6 weeks to 6 years of age unless use of the pertussis component is contraindicated. Tdap is the recommended vaccine for adolescents and adults, those over the age of 11 years.

The current immunization for tetanus and diphtheria toxoids and pertussis, Tdap, is administered to people in which age range? Any age range In the first 2 years of life Younger than 6 years of age 11 years of age and older

Family Although family dysfunctions can result from long-term stress, this would not be a manifestation of a seizure. Autonomic dysfunctions (A) can be evident in a number of body systems, including the respiratory system (affecting breathing) and the cardiac system (affecting heart rate). Motor dysfunctions (C) are the common manifestations seen with seizure disorders such as the tonic-clonic type. Sensory dysfunctions (D) are manifestations such as the aura preceding a seizure that is often described by patients.

The nurse manager of a neuro-medical surgical unit reviewing potential manifestations of seizures with an orientee would become concerned if the new nurse included which of the following dysfunctions as a manifestation? Autonomic Family Motor Sensory

"Soreness at the injection site is a common reaction." Soreness at the injection site is a common adverse effect of tetanus toxoid.

What teaching would the nurse provide to a client receiving tetanus toxoid? "Soreness at the injection site is a common reaction." "This medication must be repeated weekly for 4 weeks." "Increase fluid and fiber in your diet to prevent constipation." "You will have lifetime immunity from this injection."

intracranial regulation

mechanisms or conditions that impact the processing and function of the components of the skull

osmotic diuretics

class of medication that draws water across the BBB, leading to a decrease in interstitial volume and a subsequent decrease in ICP; mannitol is the agent used most commonly

active acquired immunity

immunity that develops after the introduction of a foreign antigen resulting in the formation of antibodies or sensitized T lymphocytes

passive acquired immunity

immunity that occurs by the introduction of preformed antibodies - either from an artificial route (transfusion) or from a natural route (breastfeeding)

extravasation

the movement of white blood cells from the capillaries to the tissues surrounding them

sepsis

the presence of pathogens in the blood or other tissues throughout the body

ferilitization

the union of a human egg and sperm, usually occurring in the ampulla of the fallopian tube

Natural passive immunization

type of immunization in which antibodies are transferred from the mother to her infant in breast milk or through the bloodstream via the placenta during pregnancy

antiserum

A serum that contains antibodies. It is usually obtained from an animal that has been immunized against a specific antigen.

sexual behavior

How one responds to sexual impulses

Second-third trimester Second and third trimester complications include anencephalus, chromosomal anomalies, gestational diabetes, group B strep, cystitis, pyelonephritis, cholecystitis, hypertension, preeclampsia, oligohydramnios, polyhydramnios, and premature rupture of membranes, etc. Leaking of clear fluid from the vagina with back pain and elevated BP is associated with premature rupture of membranes, a second trimester complication of pregnancy. Preconception is prior to becoming pregnant; postpartum is after delivery of the infant; and first trimester is not associated with premature rupture of the membranes usually.

Many females experience problems achieving and maintaining a pregnancy. The ER nursing assessment of a child-bearing-age female shows back pain, elevated blood pressure, and leaking of clear fluid from the vagina. Maternal-fetal complications described above are most often associated with which child-bearing stage? Preconception First trimester Second-third trimester Postpartum

gender identity

Perception of one's gender as masculine or feminine

cell-mediated immune system

The immune response that is mediated by T cells (as opposed to B cells, which produce antibodies). T cells mount their immune response through activities such as the release of cytokines (chemicals that stimulate other protective immune functions) as well as through direct cytotoxicity (e.g., phagocytosis of an antigen).

Exaggerated immune response MS is an autoimmune disease, which is a form of exaggerated immune response. MS is not a problem of immunodeficiency, nor is it an optimal immune response.

The nurse is caring for a patient with a diagnosis of multiple sclerosis (MS). The nurse should be aware of which associated response? Primary immunodeficiency Secondary immunodeficiency Optimal immune response Exaggerated immune response

control inflammation. Medications for RA are intended to control the inflammation that results from the body's hyperimmune response. Autoimmune diseases like RA are chronic and currently have no curative treatments. Autoimmune diseases like RA are caused by hyperimmune response. The immune system needs to be suppressed, not enhanced. While the medications used for RA might help with pain management, the goal of medication intervention is to manage the inflammation.

The nurse is preparing to administer medications to a patient with rheumatoid arthritis (RA). The nurse explains to the patient that the goal of medication treatments for RA is to eradicate the disease. enhance immune response. control inflammation. manage pain.

Confirms the nonspecific presence of inflammation An elevated ESR is indicative of the presence of inflammation in the body. Proteins produced during the inflammatory process adhere to red blood cells, causing them to be heavier and settle out of blood samples at a faster rate than normal. The ESR does not identify specific causes of inflammation and does not determine a specific location of inflammation. The ESR is a nonspecific indicator of inflammation.

The nurse is reviewing the erythrocyte sedimentation rate (ESR) of a patient to determine which significant finding? Determines specific causes of inflammation Identifies the location of inflammation within the body Confirms the nonspecific presence of inflammation Indicates a diagnosis of systemic lupus

antihypertensive Anti-hypertensive medications may be detrimental because the mean arterial pressure must be adequate to maintain cerebral blood flow and limit secondary injury. Fever can worsen the outcome after a stroke, and antipyretics can promote normothermia. Osmotic diuretics such as mannitol can decrease interstitial volume and decrease intracranial pressure. Short-acting sedatives can decrease intracranial pressure by reducing metabolic demand. Long-acting sedatives would be avoided to provide times for periodic neurologic assessments.

The nurse preparing to care for a patient after a suspected stroke would question an order for a(n) antihypertensive. antipyretic. osmotic diuretic. sedative.

venom

a poison that is secreted by an animal (e.g., snake, insect, or spider)

encephalitis

inflammation of the brain indicated by ↓LOC, seizures, and weakness; a severe adverse effect of vaccine administration

fundal height

the distance from the pubic bone to the top of the uterus measured in centimeters

intracranial pressure (ICP)

the sum of the pressure exerted by the three components in the skull

Immediate tobacco cessation Psychosocial factors affecting pregnancy include smoking, excessive use of caffeine, alcohol and drug abuse, psychological status including impaired mental health, an addictive lifestyle, spouse abuse, and noncompliance with cultural norms. Immediate tobacco cessation would be the highest priority because continued smoking could be teratogenic if the woman should become pregnant. Smoking causes vasoconstriction which restricts the amount of oxygen and nutrients to the rapidly growing fetus. Daily exercise promotes health but would not be the highest priority among these factors. Stopping caffeine and avoiding sweets are important and can be addressed after tobacco cessation.

A 20-year-old woman comes for preconceptual counseling. She wants to get pregnant soon. Which of the following health-promoting habits would have the highest priority at this time? Immediate tobacco cessation Getting daily exercise Stopping all caffeine Avoidance of sweets

"For women, the recommended age for this vaccine is 13 to 26 years of age." It is important to make sure that a patient receiving Gardasil is not pregnant and that the patient knows that there are a total of three injections, but this particular patient is too old to receive the vaccine. The guidelines recommend the vaccine for women 13 to 26 years of age.

A 30-year-old woman is in the clinic for her yearly gynecologic exam and asks the nurse about the "new vaccine that prevents HPV." She wants to receive the papillomavirus vaccine (Gardasil). Which response by the nurse is most appropriate? "For women, the recommended age for this vaccine is 13 to 26 years of age." "We will need to make sure you are not pregnant first." "There will be a total of three injections." "I will check with your health care provider and then get the first dose of the vaccine ready."

Serum sickness Serum sickness sometimes occurs after repeated injections of equine (horse)-made immunizing drugs and is characterized by edema of the face, tongue, and throat; rash; urticaria; fever; flushing; dyspnea; and other symptoms.

A 45-year-old man has received a series of immunizing drugs in preparation for a trip to a developing country. Within hours, his wife brings him to the emergency department because he has developed edema of the face, tongue, and throat and is having trouble breathing. The nurse suspects that, based on the patient's history and symptoms, he is experiencing which condition? Serum sickness Cross-sensitivity Thrombocytopenic purpura Adenopathy

The unwashed math blocks The boy has the flu and sneezes into his hand while at school. When he works with the math blocks, he leaves the flu virus on the toys. The teacher's helper picks up the virus with the blocks. When the parent touches her nose with her hand, the virus enters the susceptible host. The blocks act as the mode of transmission. The boy carries the pathogen, and his sneeze is the portal of exit. The teacher's helper is the susceptible host. The hand-to-nose contact is the portal of entry.

A 5-year-old boy with early flu symptoms is at school working with some math blocks. He sneezes into his hand and then continues working with his blocks. An unvaccinated teacher's helper cleans up the blocks when the child leaves them on the table. After touching the blocks, she rubs her nose with her hand. Which represents the most likely mode of transmission? The 5-year-old boy The unvaccinated teacher's helper The hand-to-nose contact The unwashed math blocks

excitement, plateau, and orgasmic. During these three phases, heart rate, blood pressure, and respirations increase steadily, increasing stress to the heart muscle. This would be the period of greatest concern for a patient who has recently experienced an MI. The plateau and orgasmic phases may be of physiologic concern to this patient, but during the resolution phase, vital signs return to normal, and muscles relax. Arousal is not a phase in the four-stage model of the human sexual response cycle, although some researchers feel this should be added.

A 55-year-old male patient post-myocardial infarction (MI) queries the nurse caring for him whether he will be healthy enough for sexual activity after discharge from the hospital. The patient has been prescribed anti-hypertensives and beta-blockers. While health teaching, the nurse understands that the three phases of the four-stage model of the human sexual response cycle that are of concern for this patient include excitement, plateau, and orgasmic. plateau, orgasmic, and resolution. excitement, orgasmic, and resolution. arousal, excitement, and plateau.

Tendency to allergic reactions Duration of signs and symptoms Recent exposure to poison oak or ivy It is important to know whether the signs and symptoms are related to a history of allergies, a communicable infection contracted during the trip, or some other factor. The nurse must gather information regarding the duration of signs and symptoms because they could be related to a variety of factors that may or may not be linked to the camping trip. It is important to determine whether the child was exposed to a known allergen so appropriate treatment may be initiated. It is not necessary to know when the child is expected back in school; this information is unrelated to the situation. The child's problem is also unrelated to sports activities.

A child who recently returned from a three-day camping trip over spring vacation is brought to the clinic after a rash, chills, and low-grade fever develop. What are the most important data for the nurse to assess when taking the child's history? Select all that apply. Date of return to school Sports played on camping trip Tendency to allergic reactions Duration of signs and symptoms Recent exposure to poison oak or ivy

Motor response to verbal commands The three areas of assessment to determine the level of consciousness using the Glasgow Coma Scale are motor response to verbal commands, eye opening in response to speech, and verbal response to speech. Assessing breathing patterns, deep tendon reflexes, and eye accommodation are not included in the Glasgow Coma Scale.

A client comes into the emergency department with neurologic deficits after falling off a ladder. Which client assessment will the nurse perform for the Glasgow Coma Scale? Breathing patterns Deep tendon reflexes Eye accommodation to light Motor response to verbal commands

"Were you aware of anything different or unusual just before your seizure began?" Identification of a sensation that occurs before each seizure is helpful in identifying the cause of the seizure and planning how to identify and avoid a future seizure. Although the response "Is your job demanding or stressful most of the time?" may provide some information, it is not the most inclusive question the nurse can ask; also, it limits the client's reply. Although the response "Do you participate in any strenuous sports activities on a regular basis?" may provide some information, it is not the most inclusive question the nurse can ask; also, it limits the client's reply. Although the response "Does anyone in your family have a history of central nervous system problems?" may provide some information, it is not the most inclusive question the nurse can ask; also, it limits the client's reply.

A client has a tonic-clonic seizure at work and is admitted to the emergency department. Which question is most useful when planning nursing care related to the client's seizure? "Is your job demanding or stressful most of the time?" "Do you participate in any strenuous sports activities on a regular basis?" "Does anyone in your family have a history of central nervous system problems?" "Were you aware of anything different or unusual just before your seizure began?"

Monitoring the duration and intensity of the contractions Oxytocin increases the intensity and duration of contractions; prolonged (tetanic) contractions will jeopardize the safety of the fetus and necessitate discontinuation of the drug. A bulging perineum indicates that cervical dilation is complete and birth is imminent; because cervical dilation is only 2 to 3 cm here, a bulging perineum is not expected. Documenting the fetal heart rate and its variations is important throughout labor. There is no indication at this time that a cesarean birth is necessary.

A client is admitted to the birthing unit in active labor. Cervical dilation has progressed from 2 to 3 cm over the previous 8 hours. The primary healthcare provider determines that the client has hypotonic dystocia, and an infusion of oxytocin is prescribed to augment her contractions. What is the most important nursing action at this time? Checking the perineum for bulging Documenting the fetal heart rate and its variations Preparing the client for an emergency cesarean birth Monitoring the duration and intensity of the contractions

Restlessness with dyspnea The client is at risk for airway obstruction; restlessness and dyspnea indicate hypoxia. Cloudy drainage may indicate infection, which is not an immediate postoperative complication. Loss of the gag reflex is unimportant. The pharyngeal opening is sutured closed, and a tracheal stoma is formed; the trachea is anatomically separate from the esophagus. Decreased urinary output needs to be monitored but does not take priority.

A client returns from a radical neck dissection with two portable wound drainage systems at the operative site. Inspection of the neck incision reveals moderate edema of the tissues. Which assessment finding is a priority requiring immediate nursing intervention? Cloudy wound drainage Absence of the gag reflex Decreased urinary output Restlessness with dyspnea

January 12 January 12 is the EDB. Using Nägele's rule, add 3 months and subtract 7 days from the client's last menstrual period. January 5, January 19, and January 26 all represent inaccurate applications of Nägele's rule.

A client visits the prenatal clinic because her menstrual period is late. Her last period was April 5. Testing confirms that she is pregnant. According to Nägele's rule, what date should the nurse provide as the expected date of birth (EDB)? January 5 January 12 January 19 January 26

Taking a hot tub bath or shower in the morning Moist heat increases circulation and decreases muscle tension, which help relieve chronic stiffness. Although wearing loose but warm clothing is advisable for someone with arthritis, it does not relieve morning stiffness. Inactivity promotes stiffness. The practice of avoiding excessive physical stress and fatigue is related to muscle fatigue, not to stiffness of joints.

A client with rheumatoid arthritis asks the nurse about ways to decrease morning stiffness. What should the nurse suggest? Wearing loose but warm clothing Planning a short rest break periodically Avoiding excessive physical stress and fatigue Taking a hot tub bath or shower in the morning

Oral contraceptive pills Oral contraceptive pills prevent ovulation and are 99% effective in preventing pregnancy when taken as directed. Intrauterine devices, coitus interruptus, and natural family planning will not prevent ovulation while reaching 99% effectiveness in preventing pregnancy,so they are not recommended for this college student.

A female college student is planning to become sexually active. She is considering birth control options and desires a method in which ovulation will be prevented. To prevent ovulation while reaching 99% effectiveness in preventing pregnancy, which option should be given the strongest consideration? Intrauterine device Coitus interruptus Natural family planning Oral contraceptive pills

Estrogen deficiency prevents the ovum from reaching the uterus and may be a factor in infertility. The cilia in the tubes are stimulated by high estrogen levels, which propel the ovum toward the uterus. Without estrogen, the ovum won't reach the uterus. The results of a series of events occurring in the ovary cause an expulsion of the oocyte from the ovarian follicle known as ovulation. The ovarian cycle is driven by multiple important hormones: 1) gonadotropic hormone, 2) follicle stimulating hormone (FSH), and 3) luteinizing hormone (LH). The cilia in the tubes are stimulated by high 4) estrogen levels, which propel the ovum toward the uterus. The zona pellucida (inner layer) and corona radiata (outer layer) form protective layers around the ovum. If an ovum is not fertilized within 24 hours of ovulation by a sperm, it is usually reabsorbed into a woman's body. A patient who is hypoestrogenic would not have excess circulating estrogen. A patient with low estrogen would not be classified as hyperestrogenic. Without sufficient estrogen, there can be no fertilization of the ovum.

A female infertility patient is found to be hypoestrogenic at the preconceptual clinic visit. She asks the nurse why she has never been able to get pregnant. Which response is best? Circulating estrogen contributes to secondary sex characteristics. Estrogen deficiency prevents the ovum from reaching the uterus and may be a factor in infertility. Hyperestrogen may be preventing the zona pellucida from forming an ovum protective layer. The corona radiata is preventing fertilization of the ovum.

Drinking alcohol should be avoided during pregnancy because of its teratogenic effects. Stages of development include the ovum, the embryo, and the fetus. The embryonic period lasts from the beginning of the fourth week to the end of the eighth week of gestation. Teratogenicity is a major concern because all external and internal structures are developing in the embryonic period. During pregnancy, a woman should avoid exposure to all potential toxins, especially alcohol, tobacco, radiation, and infectious agents. Living beneath power line or near an oil field is not teratogenic in itself. Stopping sweets can be addressed after the alcohol cessation is addressed.

A female patient comes to the clinic at 8 weeks' gestation. She lives in a house beneath electrical power lines, which is located near an oil field. She drinks two caffeinated beverages a day, is a daily beer drinker, and has not stopped eating sweets. She takes a multivitamin and exercises daily. She denies drug use. Which finding in the history has the greatest implication for this patient's plan of care? Electrical power lines are a potential hazard to the woman and her fetus. Living near an oil field may mean the water supply is polluted. Drinking alcohol should be avoided during pregnancy because of its teratogenic effects. Eating sweets may cause gestational diabetes or miscarriage.

At fertilization, chromosomal sex is established. Remember that the primary spermatocyte contains two sex chromosomes, one X chromosome and one Y chromosome, and the primary oocyte contains two sex chromosomes, both X chromosomes. During the first reduction division, two secondary spermatocytes are produced, one X and one Y, establishing X and Y cell lines. The X-bearing cell line is established during oogenesis. Female gametes will all be X bearing and male gametes will be either X or Y bearing. A female develops through the fertilization of the ovum by an X-bearing sperm producing an XX zygote; a male is produced through the fertilization of a Y-bearing sperm producing an XY zygote. Therefore, at fertilization, chromosomal sex is established. Chromosomal sex is not established at ovulation, ejaculation, or climax, so these choices are erroneous.

A married couple present to the preconceptual clinic with questions about how a fetus's chromosomal sex is established. What is the nurse's best response? At ovulation, chromosomal sex is established. At ejaculation, chromosomal sex is established. At climax, chromosomal sex is established. At fertilization, chromosomal sex is established.

Women who have undergone hysterectomy no longer desire to be sexually active. Women who perceive themselves as being attractive and having accessibility to a healthy partner often maintain a healthy sex life during their middle years. For many women, lack of lubrication is frequently cited as the cause for sexual problems as they reach middle age. A number of water-soluble lubricants are now readily available to provide relief for the pain that occurs during intercourse due to vaginal dryness. Hormone replacement therapy (HRT) is often used to treat the symptoms of menopause, most notably hot flashes and vaginal dryness. Hot flashes however are not known to be a main cause of decreased sexual interest. Taking HRT is not known to eliminate climactic experience during sex.

A nurse is assessing a menopausal female and discussing sexuality. Which statement is accurate regarding physiological effects of menopause on sexual health? Decreased lubrication is frequently cited as the cause for sexual problems. Women who have undergone hysterectomy no longer desire to be sexually active. Hot flashes are often bothersome and lead to decreased sexual interest. Women taking hormone replacement therapy may not experience climax during sex.

By telling the adolescent to shrug the shoulders The accessory nerve (cranial nerve XI) innervates the sternocleidomastoid and trapezius muscles; the nurse evaluates this nerve by asking the client to shrug the shoulders. The glossopharyngeal nerve (cranial nerve IX) is assessed by stimulating the pharynx with a tongue blade. The vagus nerve (cranial nerve X) controls muscles of the larynx and is assessed by asking the client to swallow. Stroking the plantar surface of the foot is a test for the presence or absence of the Babinski reflex; this test is not used for assessment of a cranial nerve.

A nurse is performing a neurologic assessment of an adolescent with a seizure disorder. How should the nurse test cranial nerve XI? By checking the gag reflex By asking the adolescent to swallow By stroking the plantar surface of the foot By telling the adolescent to shrug the shoulders

6 weeks The first dose of the series of three injections is given at 6 weeks of age. The other options are incorrect.

A nurse is working in an immunization clinic. A new colleague asks, "When is the first dose of the diphtheria, tetanus, and acellular pertussis (DtaP, Daptacel) given?" The nurse knows that this series is started at what age? At birth 6 weeks 3 months 1 year

vasodilation The inflammatory process results in rubor, or redness, of an area of insult. The body responds to injury by increasing the blood flow to an area through vasodilation. This allows increased oxygen and more nutrients and appropriate white blood cells to reach the area, isolating the area and beginning the immune response. Extravasation is the movement of fluid from its confined space into the surrounding tissue. Neutrophils are one of the most common types of white blood cells. Exudate is the fluid filled with proteins and white blood cells that moves out of the vascular spaces through extravasation.

A patient admitted to an acute care floor has rubor of an area of injury on the left lower extremity. The nurse understands that this redness is caused by vasodilation. extravasation. neutrophils. exudate.

"This test will help to better determine where the injury actually is and how severe it is." Radiographic imaging studies such as CT scans help to determine the location and extent of inflammation within the body. The CT scan will help with diagnosis. The diagnosis is not predetermined. CT scanning does not alleviate pain. Radiography does not necessarily determine a cause of an injury.

A patient comes to a clinic with a chief complaint of, "My left arm is red and swollen. It hurts badly enough that I couldn't go to work today." The physician orders computer-assisted tomography (CT) scanning of the left upper extremity. The nurse knows the patient understands the reason for the procedure when he states "I need to have this done because my arm is broken." "The doctor wants me to have this so that the pain will stop." "This will tell you what I did to my elbow because I really don't know what happened." "This test will help to better determine where the injury actually is and how severe it is."

avoid sharing razors and other personal items. Avoidance of sharing personal items like razors and hairbrushes can decrease the spread of pathogens that cause inflammation and infection. Not eating with the others in his college apartment won't relieve or prevent the spread of infection. A CBC monthly will not treat or prevent inflammation. Showers should be disinfected before and after each use.

A patient comes to the clinic with a complaint of painful, itchy feet. On interview, the patient tells the nurse that he is a college student living in a dormitory apartment that he shares with five other students. The nurse plans to teach the patient to not eat with the other students. avoid sharing razors and other personal items. have his CBC checked monthly. disinfect showers and bathroom floors weekly after use.

use of high dose systemic steroids in the past month. Varicella vaccine is not to be given to immunodeficient patients or to patients who have received high doses of systemic steroids in the previous month. It is also recommended that salicylates be avoided for 6 weeks after administration of varicella vaccine because of the possibility of Reye's syndrome.

A patient has an order for the varicella vaccine. It is most important for the nurse to assess the patient for: use of high dose systemic steroids in the past month. allergy to aspirin. allergy to eggs. history of hypertension.

helps to kill the infection causing the inflammation. Antimicrobials treat the underlying cause of the infection which leads to inflammation. Analgesics and nonsteroidal antiinflammatory drugs (NSAIDs) help to treat pain. NSAIDs and other antipyretics are cyclooxygenase inhibitors. Antipyretics help to reduce fever.

A patient is being treated with an antibiotic. The nurse explains to the patient that this medication is required for the reduction of inflammation at the injury site because this medication will decrease the pain at the site. helps to kill the infection causing the inflammation. inhibits cyclooxygenase. will reduce the patient's fever.

ice should be applied for 15 to 20 minutes every 2 to 3 hours over the next 1 to 2 days. Ice is used on areas of injury during the first 24 to 48 hours after the injury occurs to prevent damage to surrounding tissues from excessive inflammation. Ice should be used for a maximum of 20 minutes at a time every 2 to 3 hours. Ice must be used according to a schedule for it to be effective and not be overused. Using ice more often or for longer periods of time can cause additional tissue damage. Ice is recommended to inhibit the inflammatory process from damaging surrounding tissue.

A patient is diagnosed with a sprain to her right ankle after a fall. The patient asks the nurse about using ice on her injured ankle. The nurse should tell the patient that she should use ice only when the ankle hurts. ice should be applied for 15 to 20 minutes every 2 to 3 hours over the next 1 to 2 days. she should wrap an ice pack around the injured ankle for the next 24 to 48 hours. ice is not recommended for use on the sprain because it would inhibit the inflammatory response.

Help to ensure adequate social interaction and support. Frequently, patients in contact isolation do experience a decrease in social interaction because of the isolation. The nurse must help provide adequate social stimulation for the patient. Frequently, this is done by educating the family and friends regarding isolation practices. Isolation does not mean that the patient cannot have visitors. Visitors must be educated on how to maintain the contact isolation while with the patient, especially hygiene guidelines. Personal protective equipment must be used when entering the room of a patient in contact isolation. Nurses and visitors do not always know when they will come into contact with a pathogen, especially if it is highly virulent. The patient in contact isolation should have regular face-to-face contact with the nurse. The nurse should not use the call light system to communicate with a patient in isolation any more than any other patient.

A patient is in contact isolation for a bacterial infection. The nurse is going to implement which of the following interventions for this patient? Prevent all visitors from entering the room at any time during hospitalization. Use personal protective equipment only when knowingly coming into contact with pathogens. Help to ensure adequate social interaction and support. Communicate with the patient over the call light whenever possible.

Antivenins or antisera Antivenins, also known as antisera, are used to prevent or minimize the effects of poisoning by poisonous snakes and spiders. They provide the person who has been bitten with the substance needed to overcome the effects of the venom.

A patient is in the urgent care center after experiencing a black widow spider bite. The nurse prepares to give which product to treat this injury? Live vaccine Antivenins or antisera Tetanus immune globulin Active immunizing drug

A complement cascade A complement cascade is responsible for the dilation of blood vessels and leaking of fluid from the vascular system to the area of insult, resulting in the swelling and redness associated with an inflammatory response. An allergic reaction can cause edema and erythema, but the question does not provide enough information to determine the specific cause of the swelling and redness. IgE is a specific immunoglobulin associated with signs and symptoms of allergic rhinitis. Clonal diversity refers to the maturation process of cells.

A patient presents to the clinic with observable edema and erythema of the left forearm. A brief history reveals no exposure to potential irritating agents. On palpation, the nurse finds the area very warm and tender. What is the most likely cause of the patient's symptoms? An allergic reaction A complement cascade IgE reactions Clonal diversity

Tetanus immunoglobulin Vaccines and toxoids rely on a person's immune system to work. Patients who are immunocompromised (as in those receiving immunosuppressive therapy and those with AIDS) may not benefit from vaccines or toxoids because they are unable to mount an immune response. In these situations, passive immunizing drugs such as immunoglobulins are warranted.

A patient with AIDS was cut by a rusty piece of metal while walking outside. He recalls that his last tetanus booster was more than 20 years ago. Which immunization therapy will he receive at this time? He cannot receive any type of immunization therapy. Tetanus immunoglobulin Tetanus toxoid, adsorbed Tetanus and diphtheria toxoid (Td) booster

Hepatitis B immunoglobulin (BayHep B) Recombivax HB promotes active immunity to hepatitis B infection in people who are considered to be at high risk for potential exposure to the virus, whereas hepatitis B immunoglobulin provides passive immunity for the prophylaxis and postexposure treatment of people exposed to hepatitis B virus or HBs-Ag-positive materials, such as blood, plasma, or serum. Hib and Varivax vaccines are not appropriate for this situation.

A sanitation worker has experienced a needle stick by a contaminated needle that was placed in a trash can. The employee health nurse expects that which drug will be used to provide passive immunity to hepatitis B infection? Haemophilus influenzae type b (Hib) Varicella virus vaccine (Varivax) Hepatitis B immunoglobulin (BayHep B) Hepatitis B virus vaccine (inactivated) (Recombivax HB)

Painless vaginal bleeding This client's placenta is implanted near the internal cervical os; in the latter part of pregnancy, as the process of effacement occurs, placental separation from the uterus causes painless bleeding. Sharp abdominal pain occurs with abruptio placentae, the premature separation of a normally situated placenta. Increased lower back pain is not specific to a low-lying placenta. Early rupture of membranes is not specific to a low-lying placenta.

A sonogram performed on a client in the third trimester reveals a low-lying placenta. What should the nurse teach the client that she is at risk for? Sharp abdominal pain Painless vaginal bleeding Increased lower back pain Early rupture of membranes

Prostate-specific antigen (PSA) PSA testing is recommended annually for men at increased risk for prostate cancer. This includes men with a family history or those of African-American descent. HPV testing would likely be ordered for patients with genital warts. This might not be necessary for this patient. Tests for HIV should be ordered for patients that belong to high-risk populations, including men who have sex with men, and all pregnant women. All sexually active men and women should have a VDRL and rapid plasma reagin performed.

A thorough assessment of sexual health includes laboratory and other diagnostic procedures. Tests are ordered at the provider's discretion based upon gender and lifestyle of the patient. A 37-year-old heterosexual African-American man has come for his annual health screening. Which test must the nurse ensure is ordered for this patient? Human papilloma virus (HPV) Prostate-specific antigen (PSA) HIV Venereal disease research laboratory (VDRL)

passive immunization

A type of fast-acting, short-lived immunization in which immunity to infection occurs by injecting a person with or concentrated immunoglobulins that directly give the host the means to fight off an invading microorganism (artificial). The host's immune system therefore does not have to manufacture these antibodies. This process also occurs when antibodies pass from mother to infant during breastfeeding or through the placenta during pregnancy (natural).

active immunization

A type of immunization that causes development of a complete and long-lasting immunity to a certain infection through exposure of the body to the associated disease antigen; it can be natural (i.e., having the disease) or artificial (i.e., receiving a vaccine or toxoid). provides long-lasting or permanent immunity

"This must be very difficult for you with this added pressure." Stating that the situation must be difficult encourages the clients to verbalize their feelings. The clients are not seeking advice concerning their relationship with their parents; the focus should be on them. Stating that five years without a pregnancy is a long time is an insensitive statement and cuts off further communication.

After 5 years of unprotected intercourse, a childless couple comes to the fertility clinic. The husband tells the nurse that his parents have promised to make a down payment on a house for them if his wife gets pregnant this year. What is the nurse's best response to this comment? "This must be very difficult for you with this added pressure." "Having a child is a decision you should make without your parents' input." "You're lucky. It's nice that your parents are making such a generous offer." "Five years without a pregnancy is a long time. You were right to come to the fertility clinic."

An increased hospital stay An increased hospital stay increases the risk for hospital-acquired infections, which can delay wound healing. Adequate arterial blood flow improves, rather than delays, wound healing. Supplemental oxygen can increase wound healing. A healthy diet is important to wound healing.

After patient teaching, the patient is able to verbalize that which occurrence can delay wound healing after surgery? Adequate arterial blood flow to the wound Supplemental oxygen therapy A healthy diet An increased hospital stay

GCS. The GCS gives a standardized numeric score of the neurologic patient assessment. An electroencephalogram is used in diagnosing and localizing the area of seizure origin. This scale is an example of one type of specific tool for nurses to use when assessing a patient following stroke. The Monroe-Kellie doctrine is not an assessment or monitoring strategy; it describes the interrelationship of volume and compliance of the three cranial components, brain tissue, cerebral spinal fluid, and blood.

After shunt procedure, the nurse would monitor the patient's neurologic status by using the electroencephalogram. GCS. National Institutes of Health Stroke Scale. Monro-Kellie doctrine.

Using alcohol, marijuana, or illicit substances The influence of nonsexual high-risk behavior such as the use of alcohol, marijuana, and illicit substances increases sexual risk-taking behavior. The abuse of alcohol or drugs results in impaired judgment and less thoughtfulness related to the sexual act, particularly when substances are ingested close to the time of sexual activity. More varied sexual experiences and intercourse with multiple partners are significant individual sexual risk-taking behaviors. Gay, lesbian, and bisexual youth; men who have sex with men; and women who primarily have sex with women have been found to engage in more high-risk sexual practices. Youth in particular are less likely to engage in safer sex practices such as condom use. This is a sexual behavior that significantly increases the risk for contracting sexually transmitted infections, including human immunodeficiency virus (HIV) infection, and for unintended pregnancy.

Although sexual activity is considered a normative process, some individuals place themselves at increased risk for negative consequences related to this process. Which nonsexual behavior is likely to increase risk-taking activities? Having multiple sexual partners Using alcohol, marijuana, or illicit substances Having gay, lesbian, or bisexual partners Refraining from safe-sex practices such as condom use

primary immunodeficiency. Primary immunodeficiency is a risk for patients with two or more of the listed problems. Secondary immunodeficiency is induced by illness or treatment. Cancer is caused by abnormal cells that will trigger an immune response. Autoimmune diseases are caused by hyperimmunity.

An 18-month-old female patient is diagnosed with her fifth ear infection in the past 10 months. The physician notes that the child's growth rate has decreased from the 60th percentile for height and weight to the 15th percentile over that same time period. The child has been treated for thrush consistently since the third ear infection. The nurse understands that the patient is at risk for primary immunodeficiency. secondary immunodeficiency. cancer. autoimmunity.

Egg Contraindications to the administration of immunizing drugs include allergy to the immunization itself or allergy to any of its components, such as eggs or yeast.

An allergy to which substance is a contraindication to the administration of an immunizing drug? Wheat Egg Corn Soy

Fever accompanied by decreased responsiveness The most common complication of surgery would be infection as evidenced by fever and decreased responsiveness. This is true for a patient of any age. The fluid in the peritoneum would be reabsorbed; excessive accumulation is not common. The setting sun sign, or eyes with the sclera visible above the irises, is a late sign of increased intracranial pressure. The early signs are taught prior to discharge. Seizures are not a common complication of ventriculoperitoneal shunt surgery.

An infant born with hydrocephalus is to be discharged after insertion of a ventriculoperitoneal shunt. Which common complication of this type of surgery should the nurse explain to the home caregivers to prepare them for the patient's discharge? Excessive fluid accumulation in the abdomen Eyes with sclera visible above the irises Fever accompanied by decreased responsiveness Violent involuntary muscle contractions

tertiary prevention. An example of tertiary prevention relating to reproductive health would be managing fetal intrauterine growth restriction by serial ultrasounds. This type of diagnostic maternal/fetal monitoring is performed to determine the best time for delivery due to potential fetal nutritional, circulatory, or pulmonary compromise. A cesarean section (operative delivery) may be performed if maternal or fetal conditions indicate that delivery is necessary. Antenatal diagnostics refers to prior to pregnancy. An example of primary prevention is teaching a high school class about reproductive health. An example of secondary prevention is prenatal care in the second trimester of pregnancy to prevent problems for the developing fetus.

An obstetric multipara with triplets is placed on bed rest at 24 weeks' gestation. Her perinatologist is managing intrauterine growth restriction with serial ultrasounds. This is an example of antenatal diagnostics. primary prevention. secondary prevention. tertiary prevention.

verbal responsiveness. Components of the GCS include eye opening, motor responsiveness, and verbal responsiveness. The nurse would want to assess the blood pressure, but this is not a component of the coma scale. Assessment of cranial nerve function is appropriate as alterations such as cranial nerve VI palsies may occur, but this is not part of the coma scale. Increases in head circumference are associated with alterations in intracranial pressure in infants, but this is not part of the coma scale.

Components of the GCS the nurse would use to assess a patient after a head injury include blood pressure. cranial nerve function. head circumference. verbal responsiveness.

Excuse the parent. Although all of these actions are important, in this situation the parent should be excused in order to allow the teen to discuss her sexual concerns without fear of repercussions. Adolescents may be concerned about their altered appearance and impulse control. This is the ideal time to encourage the patient to ask questions and reassure her that she does not appear ignorant. All patients should be screened for possible abuse, and this is the most appropriate time to do so. If the parent remains present, the patient may be reluctant to answer a question honestly about any history of childhood sexual abuse. The nurse must pose all questions to the patient in a nonjudgmental manner and ensure her that all answers will be kept strictly confidential.

Despite the importance of sexual health to overall well-being, many nurses and patients are uncomfortable discussing issues related to sexuality. It is for this reason that the nurse must include questions regarding a sexual health history as part of a comprehensive health assessment. A 15-year-old female patient has come to the office for her annual physical and first pelvic examination. In this situation, which nursing action is most important? Encourage the patient to ask questions about sexuality. Screen for possible abuse. Excuse the parent. Ensure the patient that all information will be kept confidential.

Cervical dilation is evident. Progressive cervical dilation is the most accurate indication of true labor. With true labor, contractions will increase with activity. Contractions of true labor persist in any position. Contractions may not begin until 24 to 48 hours after the membranes rupture.

How does the nurse distinguish true labor from false labor? Cervical dilation is evident. Contractions stop when the client walks around. The client's contractions progress only when she is in a side-lying position. Contractions occur immediately after the membranes rupture.

72 hours Ovum can be fertilized up to 72 hours after its release. The ovum disintegrates after 72 hours, and menstruation begins soon after. Therefore the ovum cannot be viable for 74, 76, or 78 hours, and fertilization will not occur.

How long will a client's ovum stay viable after its release to get fertilized? 72 hours 74 hours 76 hours 78 hours

sexual orientation

How one views self in terms of emotionally, romantically, sexually, or affectionately attracted to an individual of a particular gender

Reproduction The most obvious overlap between concepts is that of sexuality and reproduction. An example may be the use of contraceptives in order to avoid pregnancy. Women who are unable to conceive a child may experience emotional distress. A sexual relationship is likely to change as pregnancy advances. If a patient is feeling stress because of other life issues, this is likely to have a negative impact on his or her sexual relationships. The patient who has poor gas exchange may encounter challenges with sexual activity related to hypoxia. One physiological barrier to healthy sexual functioning is pain. Both chronic pain and pain during intercourse can negatively affect a patient's sexual relationship.

Human sexuality is interrelated with a variety of other nursing concepts that may affect sexuality or be affected by healthy sexual functioning. Prompt diagnosis and treatment of potential concerns related to concept overlap is an important nursing function. Which other concept is most likely to overlap with sexuality? Stress Gas exchange Pain Reproduction

An individual with intellectual or developmental disabilities As more of these individuals move into mainstream society, it is important that sexual health is promoted, including teaching regarding sexual norms. Otherwise these individuals are likely victims of unhealthy sexual practices or sexual abuse. In today's society, the newly unpartnered are likely to begin dating and acquire one or more new sexual partners. This group is at significant risk for exposure to sexually transmitted infections and requires health teaching related to safer sexual practices. The Hispanic teenage girl is at increased risk for unintended teen pregnancy. Adolescent pregnancy puts an undue burden on the young woman during a crucial period of growth and development. Hispanic teens experience double the rate of pregnancy of Caucasian adolescents. Major health disparities continue to exist between African-Americans and their Caucasian counterparts—in particular a significantly increased risk for human immunodeficiency virus/acquired immunodeficiency syndrome (HIV/AIDS) and other sexually transmitted diseases.

In order to fully assess the patient and plan appropriate care including health teaching regarding sexuality, it is important for the nurse working in either a primary care or hospital setting to be cognizant that some groups of patients will have an increased risk for problems related to the concept of sexual health. Which patient is most at risk for sexual abuse? A recently divorced 50-year-old woman A Hispanic teenage girl A 30-year-old African-American male An individual with intellectual or developmental disabilities

Gender identity Gender identity is socially derived from experiences with family, friends, and society. Sexual identity is defined as whether one is male or female based on biological sexual characteristics. Sexual orientation is how one views oneself in terms of being emotionally, romantically, or sexually attracted to an individual of a particular gender. Sexual behavior is how one responds to sexual impulses and desires.

In order to fully understand the concept of sexuality, it is necessary to become familiar with the terms used when discussing this topic. Which term best describes how one views oneself as masculine or feminine? Sexual identity Sexual orientation Gender identity Sexual behavior

Parkinson's disease Parkinson's disease is an exemplar of the degenerative diseases that can affect intracranial regulation. Encephalitis is an exemplar of the inflammatory disorders that can affect intracranial regulation, as is meningitis. Brain tumors are exemplars of a third type of pathological process that can affect intracranial regulation.

Intracranial function can be disrupted by which degenerative disease of the brain? Encephalitis Meningitis Parkinson's disease Brain tumors

Traumatic brain injury Intracranial regulation would be a concern for a patient who suffered a traumatic brain injury. The primary concern for a patient with failure to thrive would be nutrition. The primary concern for a patient with an upper respiratory infection would be oxygenation. Concerns for the patient with a urinary tract infection would be thermoregulation and pain.

Intracranial regulation would be a priority concern for the nurse caring for a patient with which admitting diagnosis? Failure to thrive Traumatic brain injury Upper respiratory infection Urinary tract infection

autoregulation

Maintenance of balance to promote an environment conducive to optimal brain function

contains the materials used by the body in the initial inflammatory response. Exudate is fluid moved from the vascular spaces to the area around a wound. It contains the proteins, fluid, and white blood cells (WBCs) needed to contain possible pathogens at the site of injury. Exudate appears as part of all inflammatory responses and does not mean an infection is present. Exudate is part of normal inflammatory responses which contain self-monitoring mechanisms to help prevent damage to healthy tissue. Exudate appears at wound sites regardless of cleaning done to the area of injury.

On admission to the clinic, the nurse notes a moderate amount of serous exudate leaking from the patient's wound. The nurse realizes that this fluid contains the materials used by the body in the initial inflammatory response. indicates that the patient has an infection at the site of the wound. is destroying healthy tissue. results from ineffective cleansing of the wound area.

"Have you ever received donor semen, eggs, or transplanted tissue?" Receipt of any donated organ, tissue, semen, or eggs is considered a blood-related risk. Other blood-related risks include blood transfusion, sex with a person with hemophilia, or sharing equipment for tattoos and body piercing. The exchange of sex for money, drugs, or shelter is considered a drug use-related risk. Other drug use-related risks include having sex with a person who uses or shares, and having sex while stoned, high, or drunk so that you cannot remember the details. By using male condoms, female condoms, or other barriers, patient can protect themselves against sexual risk. Other high-risk behaviors in this category include: having sex against one's will, failing to use protection, having sex with a partner who is bisexual or gay, having anal intercourse, and sexual activity with an increased number of partners. Sharing equipment to inject street drugs or steroids is a drug use-related risk.

Preventing infection remains the most effective way of reducing the adverse consequences of sexually transmitted infections, in particular those that are not readily curable. Nurses are often able to reassure the patient enough to open dialog regarding possible exposure, testing, and treatment options. When assessing high-risk behaviors, which question specifically identifies a blood-related risk? "Have you ever received donor semen, eggs, or transplanted tissue?" "Have you ever exchanged sex for drugs, money, or shelter?" "How do you protect yourself from HIV and sexually transmitted infections?" "Have you ever injected drugs using shared equipment?"

violence prevention. Injury prevention measures such as wearing a seat belt, helmet use, firearm safety, and violence prevention programs reduce the risk of traumatic brain injuries. Blood pressure control and exercising can decrease the risk of vascular disease, impacting the cerebral arteries, rather than head injuries. Smoking cessation is one primary prevention strategy which can decrease the risk of vascular disease. Maintaining a healthy weight can decrease the risk of vascular disease.

Primary prevention strategies to reduce the occurrence of head injuries would include blood pressure control. smoking cessation. maintaining a healthy weight. violence prevention.

the male condom. When used correctly, the male condom continues to be the single most effective method for preventing sexually transmitted diseases as well as being a very highly effective contraceptive agent. A significant primary prevention strategy is the recent introduction of a vaccine used to prevent cervical cancer and genital warts caused by HPV. One of two FDA-approved vaccines should be routinely administered to 11- and 12-year-old girls and can be given up to the age of 26. HIV screening is recommended for all sexually active teens by the Centers for Disease Control and Prevention. Screening for existing disease is a secondary prevention strategy. By educating teens towards behavior change related to high-risk behaviors, nurses may be able to reduce the risk for contracting sexually transmitted infections.

Primary strategies are those that are implemented in order to avoid the development of disease. These strategies can be either population-based or individually-based. As a school nurse, you are developing a curriculum for a junior human sexuality class. In order to provide the most up-to-date information, you are aware that the single most effective primary prevention strategy for preventing sexually transmitted diseases is a vaccine to prevent HPV infection. HIV screening. education directed at high-risk behaviors. the male condom.

The best predictor of sexual health is emotional well-being. The patient with sexual dysfunction is at risk for anxiety and depression. The best predictor of sexual health is emotional well-being rather than the impairment of the physical aspects of sexual arousal and function. Nurses must remain cognizant that sexual dysfunction, regardless of the cause, is likely to result in a number of negative consequences including anxiety, stress, and depression. Although sexual arousal may be diminished by biological factors such as illness and hormone levels, psychologic factors such as anxiety, mood disorders, or stress play a more significant role in sexual health. Sexual dysfunction is more common in women, with 40 to 45% of women reporting symptoms as opposed to 20 to 30% of men. It appears that sexual dysfunction is very common among the general population, with rates varying from 20 to 50%.

Sexual dysfunction is a negative consequence of human sexuality manifested by any disturbance to the psycho-physiologic changes that occur during the sexual response cycle. Which statements related to sexual dysfunction are correct? (Select all that apply.) Biological factors play a more significant role than psychologic factors. Sexual dysfunction is more prevalent among men than women. The best predictor of sexual health is emotional well-being. The patient with sexual dysfunction is at risk for anxiety and depression. Sexual dysfunction remains uncommon.

Surgical correction Depending on the cause, a cystocele can be readily corrected by surgery. Pelvic muscle floor training (Kegel exercises) will most definitely help with symptoms of urinary incontinence. This alone is not adequate treatment for this patient. Vaginal pessaries are an excellent treatment modality for uterine prolapse. Lifestyle changes such as weight loss, avoiding constipation, and reducing high-impact exercise, such as running, will all help the patient with pelvic organ prolapse. Although these modalities will provide relief, they will not correct the cystocele without surgical intervention.

Symptoms of sexual dysfunction and altered body image often coexist with prolapse of the female reproductive organs. Nursing care requires a great deal of sensitivity, because many women are embarrassed by their condition. Your patient is a 44-year-old married woman who is complaining of painful intercourse and incontinence. Clinical evaluation reveals that the patient has a cystocele. Which treatment option is most appropriate for this patient? Pelvic floor training Vaginal pessaries Surgical correction Lifestyle changes

Veterinarians Military personnel Workers who process imported animal hair People at risk for exposure to the anthrax bacterium include military personnel, veterinarians, and workers who process imported animal hair.

The anthrax vaccine is recommended for which groups of people? (Select all that apply.) Select all that apply. Veterinarians Military personnel Emergency department health care providers Workers who process imported animal hair

change in level of consciousness. A change in level of consciousness is the earliest and most sensitive indication of a change in intracranial processing. This is assessed with the Glasgow Coma Scale (GCS), which assesses eye opening and verbal and motor response. The inability to focus may indicate a change, but it is not one of the earliest indicators or a component of the GCS. Primitive reflexes refers to those reflexes found in a normal infant that disappear with maturation. These reflexes may reappear with frontal lobe dysfunction and may be tested for with a suspected brain injury, so it would be the reappearance of primitive reflexes. A change in pupil size or unequal pupils may indicate a change, but they are not one of the earliest indicators or a component of the GCS.

The earliest and most sensitive assessment finding that would indicate an alteration in intracranial regulation would be change in level of consciousness. inability to focus visually. loss of primitive reflexes. unequal pupil size.

Oral temperature 38.6° C/101.5° F WBC 20×10⁹/L Patient reports, "I'm tired all the time. I haven't felt like myself in days." Systemic manifestations of inflammatory response include elevated temperature, leukocytosis, and malaise and fatigue. Purulent exudates and pain are both considered local manifestations of inflammation.

The nurse assesses the patient and notes all of the following. Select all of the findings that indicate the systemic manifestations of inflammation. Oral temperature 38.6° C/101.5° F Thick, green nasal discharge Patient complaint of pain at 6 on a 0 to 10 scale on palpation of frontal and maxillary sinuses WBC 20×10⁹/L Patient reports, "I'm tired all the time. I haven't felt like myself in days."

Strongly advise immediate tobacco cessation There are numerous risk factors for women and men affecting reproductive health and pregnancy outcomes. These can be categorized into biophysical, psychosocial, sociodemographic, and environmental factors. Some of the risk factors for human reproduction fit into multiple categories. Psychosocial factors cover smoking, excessive caffeine, alcohol and drug abuse, psychologic status including impaired mental health, addictive lifestyles, spouse abuse, and noncompliance with cultural norms. Drinking a cup of a caffeinated beverage a day is not associated with adverse fetal outcomes usually. Serum and urine testing for drug/alcohol use is not required for stated marijuana use in the remote past. Patient referral to a 12-step program is usually advisable for current alcohol and/or drug use.

The nurse is admitting a prenatal patient for diagnostic testing. While eliciting the psychosocial history, the nurse learns the patient smokes a pack of cigarettes daily, drinks a cup of cappuccino with breakfast, has smoked weed in the remote past, and is a social drinker. Which action should the nurse first take? Strongly advise immediate tobacco cessation Elimination of all caffeinated beverages Serum and urine testing for drug use and alcohol use Referral to a 12-step program

May kill healthy cells along with foreign antigens. Cytotoxic T lymphocytes can kill healthy tissue along with antigens. Suppressor T cells help to keep cytotoxic T cells in check. Helper T cells are the most prevalent type of T lymphocyte, not cytotoxic cells. Cytotoxic T lymphocytes do not suppress the immune response but are a factor in optimal immune functioning. Suppressor T lymphocytes help to suppress the function of cytotoxic cells. Dendritic cell function enhances cytotoxic T lymphocyte functioning.

The nurse is caring for a patient experiencing an immune response. She assesses the patient for development of a hyperimmune response while knowing that cytotoxic T cells are responsible for which action? May kill healthy cells along with foreign antigens. Are the most prevalent type of T lymphocyte. Can suppress the immune response. Diminish dendritic cell function.

Basic infection control techniques The spleen is one of the major organs of the immune system. Without the spleen, the patient is at higher risk for infection; so, the nurse must be sure that the patient understands basic principles of infection control. The patient with a splenectomy does not need to understand the mechanisms of inflammatory response. The patient with a splenectomy does not need to wear a face mask in public as long as the patient understands and maintains the basic principles of infection control. The patient who has had a splenectomy does not need to limit contact with the general population as long as the patient understands and maintains the basic principles of infection control.

The nurse is caring for a patient who is being discharged home after a splenectomy. What information on immune function needs to be included in this patient's discharge planning? The mechanisms of the inflammatory response Basic infection control techniques The importance of wearing a face mask in public Limiting contact with the general population

anaphylactic reaction. The patient is exhibiting signs and symptoms of an anaphylactic reaction to the medication. These signs and symptoms during administration of a medication do not correspond to a suppressed immune response but a type of hyperimmune response. While the patient is experiencing a hyperimmune response, the signs and symptoms allow for a more specific response. While the patient is experiencing an allergic reaction, the signs and symptoms presented in the scenario allow for a more specific response.

The nurse is caring for a patient who was started on intravenous antibiotic therapy earlier in the shift. As the second dose is being infused, the patient reports feeling dizzy and having difficulty breathing and talking. The nurse notes that the patient's respirations are 26 breaths/min with pulse 112 beats/min and weak. The nurse suspects that the patient is experiencing a(n) suppressed immune response. hyperimmune response. allergic reaction. anaphylactic reaction.

Clustering many nursing activities It is important to minimize stress and activities that could increase intracranial pressure. Combining many nursing activities could increase oxygen demand and intracranial pressure. This would not be safe. Interventions which can promote venous outflow can help decrease intracranial pressure. The stress of constipation or bowel movements can increase intracranial pressure; stool softeners or laxatives can minimize this.

The nurse is caring for a patient with increased intracranial pressure. Which action is considered unsafe? Aligning the neck with the body Clustering many nursing activities Elevating the head of the bed 30 degrees Providing stool softeners or laxatives as ordered

his immune system is functioning properly. Tissue integrity is closely associated with immunity. Openings in the integumentary system allow for the entrance of pathogens. If the immune response is functioning optimally, the body responds to the insult to the tissue by protecting the area from invasion of microorganisms and pathogens with inflammation. Routine vaccinations have no bearing on the body's response to intentional tissue impairment. The redness and swelling at the incision site in the first 12 to 24 hours is part of optimal immune functioning. A patient with erythema and edema that persist or worsen should be evaluated for infection. Suppressor T-cells help to control the immune response in the body.

The nurse is caring for a postoperative patient who had an open appendectomy. The nurse understands that this patient should have some erythema and edema at the incision site 12 to 24 hours post operation if his immune system is functioning properly. he is properly vaccinated. he has an infection. the suppressor T-cells in his body are activated.

Fibrous tissue replacing damaged tissue when injury is extensive Scar tissue, or fibrous repair of damaged tissue, occurs when an area is damaged too extensively for the body to replace damaged tissue with identically functioning tissue after removal of injurious agents and pathogens. Optimal functioning of the inflammatory process will result in regeneration of tissue that functions identically to the damaged and replaced tissue. Chronic inflammation can result in fibrous, or scar, tissue, but that scar tissue production is continuous as the inflammation continues. Fibrous tissue production can result from many different kinds of injuries, not just surgical wounds.

The nurse is completing an admission assessment of a new patient to the unit. The nurse notes a long, thin, fading scar on the patient's abdomen in the right lower quadrant. What is the best explanation for the scar's appearance? Optimal functioning of the inflammatory process after an injury Fibrous tissue replacing damaged tissue when injury is extensive The development of chronic inflammation A surgical incision

Anthrax Infection with Bacillus anthracis, the cause of anthrax, can occur via three routes of exposure: cutaneous, gastrointestinal, and inhalation. The other options are incorrect.

The nurse is conducting a class on potential bioterrorism agents, and a participant asks, "Which one has three routes of exposure to humans?" Which response by the nurse is correct? Anthrax Smallpox Botulism Tularemia

Natural passive immunization Natural passive immunization occurs when antibodies are transferred from the mother to her infant in breast milk or through the bloodstream via the placenta during pregnancy. Artificial active immunization causes an antigen-antibody response and stimulates the body's defenses to resist any subsequent exposures. Passive immunization is conferred by bypassing the host's immune system and injecting the person with antiserum or concentrated antibodies obtained from other humans or animals; this gives the host direct means of fighting off an invading microorganism. The host's immune system therefore does not have to manufacture these antibodies.

The nurse is reviewing principles of immunization. What type of immunization occurs when antibodies pass from mother to infant during breastfeeding or through the placenta during pregnancy? Artificial active immunization Attenuating immunization Natural passive immunization Artificial passive immunization

Patients with a febrile illness Patients who are immunosuppressed Those receiving cancer chemotherapy Contraindications to the administration of immunizing drugs include a history of reactions to or serious adverse effects resulting from the drugs, and patients who are already immunosuppressed (patients with AIDS and patients receiving chemotherapy). Immunizations are best deferred until after a febrile illness. Children younger than 1 year of age and the elderly may receive immunizing drugs.

The nurse is reviewing the health history of a new patient who may need immunizations. Active immunizations are usually contraindicated in which patients? (Select all that apply.) Patients with a febrile illness Children younger than 1 year of age Elderly patients Patients who are immunosuppressed Those receiving cancer chemotherapy

It is a one-time vaccine. The vaccine is recommended for patients 50 years of age and older. The vaccine is used to prevent reactivation of the zoster virus that causes shingles. Zoster vaccine (Zostavax) is used to prevent shingles; it also prevents reactivation of the zoster virus that causes shingles. It is given to patients 50 years of age and older, and it is a one-time vaccine. It is not given to prevent chickenpox or given to children. It does not prevent postherpetic neuralgia, and it can be given to patients who have already had shingles.

The nurse is reviewing the information about the herpes zoster vaccine (Zostavax) before administering the dose. Which statements about the vaccine are true? (Select all that apply.) It is a one-time vaccine. The vaccine is recommended for patients 50 years of age and older. The vaccine is given to children to prevent chickenpox. It is used to prevent postherpetic neuralgia. It is contraindicated in patients who have already had shingles. The vaccine is used to prevent reactivation of the zoster virus that causes shingles.

"Hand sanitizer works just as well as washing with soap and water." Hand sanitizer does not work as well as soap and water, because it is not effective against all pathogens or in all situations. For example, hand sanitizer should not be used when hands are visibly dirty. Repeating the song "Happy Birthday" twice takes about 20 seconds, which is how long hands should be rubbed together with soap. Hand sanitizer needs to be at least 60% alcohol to be effective. Hand washing before eating is recommended by the Centers for Disease Control.

The nurse is teaching a class of junior high school students about infection control through effective hand washing. Which statement made by a student indicates the need for further teaching? "Hand sanitizer works just as well as washing with soap and water." "If I sing the song "Happy Birthday" twice through while scrubbing my hands, that should be long enough." "I need to read the label on the hand sanitizer to be sure that it's at least 60% alcohol." "We should all wash our hands before eating lunch every day."

"I need to avoid getting infections because they will increase the immune response in my body, which can make my SLE worse." SLE is a hyperimmunity problem. Pathogens trigger the immune response in the body, which can exacerbate the SLE. Immunizations trigger the immune response in the body to help create antibodies. In patients with autoimmune diseases such as SLE, immunizations can exacerbate the disease. SLE is not the result of immunosuppression. Lifestyle changes are required with most chronic illnesses such as SLE. Patients cannot depend on medications alone.

The nurse is teaching a patient with a new diagnosis of systemic lupus erythematosus (SLE) about her disease. The nurse recognizes that the patient understands the information when making which statement? "I need to avoid getting infections because they will increase the immune response in my body, which can make my SLE worse." "I need to be sure to take all the available immunizations to keep me from getting sick." "Because of my SLE, my immune system is already diminished, so I need to avoid people with the flu." "As long as I take all my prescribed medications, I won't have to make any lifestyle changes as a result of my SLE."

Consistently taking prescribed medication Tubercle bacilli are particularly resistant to treatment and can remain dormant for prolonged periods; medication must be taken consistently as prescribed. Although getting sufficient rest, getting plenty of fresh air, and changing the current lifestyle are important, the microorganisms must be eliminated with medication.

The nurse provides discharge teaching to a client with tuberculosis. Which treatment measure does the nurse reinforce as the highest priority? Getting sufficient rest Getting plenty of fresh air Changing the current lifestyle Consistently taking prescribed medication

early stage acute Elevated neutrophils and monocytes within normal limits are findings indicative of early inflammatory response. Neutrophils increase in just a few hours, while it takes the body days to increase the monocyte levels. Chronic inflammation results in varying elevations in WBCs dependent on multiple issues. Elevated neutrophils are not indicative of resolved inflammation. Elevations in monocytes occur later in the inflammatory response.

The nurse reviews the patient's complete blood count (CBC) results and notes that the neutrophil levels are elevated, but monocytes are still within normal limits. This indicates _____ inflammatory response. chronic resolved early stage acute late stage acute

Are part of primary prevention for system disorders Help protect individuals and communities Are recommended by the Centers for Disease Control and Prevention (CDC) Immunizations are considered part of primary prevention, help protect individuals from contracting specific diseases and from spreading them to the community at large, and are recommended by the CDC. Immunizations are recommended for people at various ages from infants to older adults. Vaccination does not guarantee that the recipient won't get the disease, but it decreases the potential to contract the illness. No medication is risk free.

The parents of a newborn question the nurse about the need for vaccinations: "Why does our baby need all those shots? He's so small, and they have to cause him pain." The nurse can explain to the parents that which of the following are true about vaccinations? (Select all that apply.) Are only required for infants Are part of primary prevention for system disorders Prevent the child from getting childhood diseases Help protect individuals and communities Are risk free Are recommended by the Centers for Disease Control and Prevention (CDC)

To prevent reactivation of the zoster virus that causes shingles in patients age 60 years and older Zoster vaccine (Zostavax) is a vaccine for the prevention of herpes zoster. The vaccine is recommended for patients 60 years and older to prevent reactivation of the zoster virus that causes shingles. The vaccine does not prevent postherpetic neuralgia. It can be given to patients who have already had shingles. The vaccine should not be used for the prevention of chickenpox and should not be given to children.

The zoster vaccine (Zostavax) is used in which situation? To prevent chickenpox in children To prevent chickenpox in children who have been exposed to herpes zoster To prevent postherpetic neuralgia in patients who have shingles To prevent reactivation of the zoster virus that causes shingles in patients age 60 years and older

Haemophilus influenzae type b (Hib) vaccine H. influenzae type b conjugate vaccine is usually given to patients with one of the following disorders: sickle cell anemia, an immunodeficiency syndrome, Hodgkin's disease, and others. The other options are incorrect.

Two patients arrive at the clinic; one is a young boy with sickle cell anemia, and another is a 57-year-old woman with early stages of Hodgkin's disease. The nurse notices that both patients need the same vaccine. What vaccine would that be? Varicella virus vaccine (Varivax) Herpes zoster vaccine (Zostavax) Hepatitis B virus vaccine, inactivated (Recombivax HB) Haemophilus influenzae type b (Hib) vaccine

Haemophilus influenzae type b (Hib) vaccine H. influenzae type b conjugate vaccine is usually given to patients with one of these disorders: sickle cell anemia, an immunodeficiency syndrome, Hodgkin's disease, and others. The other options are incorrect.

Two patients arrive at the clinic; one is a young boy with sickle cell anemia, and another is a 57-year-old woman with early stages of Hodgkin's disease. The nurse notices that both patients need the same vaccine. What vaccine would that be? Varicella virus vaccine (Varivax) Herpes zoster vaccine (Zostavax) Hepatitis B virus vaccine, inactivated (Recombivax HB) Haemophilus influenzae type b (Hib) vaccine

Routines provide stability for clients with dementia. Rituals and routines in activities of daily living provide a framework and structure for clients with dementia, adding to their sense of safety and security. Touch is a universal message that denotes caring; it can be soothing and will not encourage dependency. Regressive behavior under stress has a calming effect and should be allowed. Care should be explained to all clients; simple declarative statements are usually understood.

What should a nurse who is caring for a hospitalized older client with dementia consider before planning care? Physical contact will increase dependency needs. Routines provide stability for clients with dementia. Regressive behavior should be interrupted immediately. Procedures do not have to be explained to clients with dementia.

Vomiting Symptoms of increased intracranial pressure include headache, decreased consciousness, and vomiting without nausea. Signs may include cranial nerve VI palsies, papilledema, periorbital bruising, and the late sign of Cushing's triad. Cerebral or brain parenchymal edema rather than dehydration occurs for many reasons. There may be chewing or swallowing problems, but hunger is not associated with increased intracranial pressure. One of the key signs of increasing intracranial pressure is vomiting occurring without nausea.

When caring for a patient on the neuro-trauma unit, the nurse should assess for which signs and symptoms of increased intracranial pressure? Dehydration Hunger Nausea Vomiting

hypertension, and bradycardia. Hypertension with widening pulse pressure, bradycardia, and respiratory changes are the ominous late signs of increased intracranial pressure and indications of impending herniation (Cushing's triad). It is bradycardia, not tachycardia, which is the component of this ominous triad. It is hypertension, not hypotension, which is the component of this ominous triad.

When caring for the patient after a head injury, the nurse would be most concerned with assessment findings which included respiratory changes, hypertension, and bradycardia. hypertension, and tachycardia. hypotension, and bradycardia. hypotension, and tachycardia.

A client who shares equipment to snort or smoke drugs Clients who use equipment to snort (straws) and smoke (pipes) drugs are at the highest risk for becoming infected with HIV as their judgment may be impaired regarding the high-risk behaviors. Safe activities that prevent the risk of contracting HIV include mutual masturbation, masturbation, and other activities that meet the "no contact" requirements. A client who undergoes perinatal HIV voluntary testing may reduce the chances of getting infected. Insertive sex between partners who are not infected with HIV are not at risk of becoming infected with HIV.

Which client has the highest risk for human immunodeficiency virus (HIV) infection? A client who is involved in mutual masturbation A client who undergoes voluntary prenatal HIV testing A client who shares equipment to snort or smoke drugs A client who engages in insertive sex with a non-infective partner

A patient whose lab results reveal leukopenia The patient with a decrease in the number of white blood cells (leukopenia) is at greatest risk for contracting a primary infection because of a weakened primary defense system. A patient with a diagnosis of diabetes mellitus is at greater risk for infection than a patient who does not have the disease but does not have the greatest risk of the four patients described. The patient receiving broad-spectrum antibiotics already has an infection and is at risk for a secondary infection. The patient who has undergone a surgical procedure is at risk for a bacterial infection but does not have the greatest risk of the patients described. Laparoscopy minimizes invasion and tissue impairment.

Which of the following patients is at greatest risk for contracting a primary bacterial infection? A patient with newly diagnosed diabetes mellitus A patient whose lab results reveal leukopenia A patient receiving broad-spectrum antibiotics A patient following laparoscopic cholecystectomy

79-year-old man with diabetes The 79-year-old man is at highest risk for inflammatory reactions among these patients for two reasons, his age and having diabetes. The risk would be high during the first year of life, but this 2-year-old girl has gotten beyond this risk period and she also has the positive factor of a healthy diet. The 38-year-old man is not in a high-risk category because of age but is because of obesity. Although a 54-year-old woman is getting older, being in menopause does not increase the risk for inflammatory reactions.

Which of the following patients is at higher risk for inflammatory reactions? 2-year-old girl with a healthy diet 38-year-old man who is obese 54-year-old woman in menopause 79-year-old man with diabetes

Cognition Mobility Oxygenation Perfusion Cognition, mobility, oxygenation, and perfusion have the strongest links to intracranial regulation and include processes that are essential for the nurse to consider when caring for a patient with intracranial concerns. Cognitive function is dependent on an optimally functioning brain. Mobility is frequently affected by intracranial regulation problems, with the most common example being a cerebrovascular accident. Perfusion and oxygenation are intimately involved with intracranial regulation, and without adequate perfusion and oxygenation, the brain cannot function. Other processes that may be closely related include clotting and pain, and interpersonal violence may also be a consideration. Safety (E) refers to the prevention of injuries or freedom from accidents, both of which could be related to intracranial regulation but would not be the strongest links for the nurse to consider.

Which of the following processes have the strongest links to intracranial regulation? Select all that apply. Cognition Mobility Oxygenation Perfusion Safety

Diencephalon The thalamus is considered to be the major relay station or "central switchboard" for the central nervous system (CNS). The thalamus, along with the hypothalamus and epithalamus, are located in the diencephalon of the brain. The cerebrum is the largest part of the brain, which has right and left lateral ventricles deep inside and has basal ganglia at its base. The brainstem connects the rest of the brain with the CNS. It is associated with life support and basic functions, such as movement. The cerebellum is concerned with coordination of movement and works together with the brainstem to focus on the functionality of the muscles. This structure is found below the occipital lobe and adjacent to the brainstem.

Which part of the brain contains the client's "central switchboard" of the central nervous system? Cerebrum Brain stem Cerebellum Diencephalon

Brain stem The brainstem, which connects the brain to the CNS, is concerned primarily with life support and basic functions, such as breathing and movement. The cerebrum controls intelligence, creativity, and memory. The cerebellum is concerned with coordination of movement. The cerebral cortex is part of the cerebrum, which is involved with almost all of the higher functions of the brain.

Which part of the client's brain is primarily associated with life support and basic functions of the body? Cerebrum Brain stem Cerebellum Cerebral cortex

Vaccines work by stimulating the humoral immune system. Vaccines work by stimulating the humoral immune system, which synthesizes immunoglobulins. They also stimulate the formation of antibodies against their specific antigen, providing active immunity.

Which statement MOST accurately describes the pharmacodynamics of vaccines? Vaccines work by suppressing the amino acid immunoglobulin sequence. Vaccines provide IgG antibodies to protect against infection. Vaccines work by stimulating the humoral immune system. Vaccines prevent the formation of antibodies against a specific antigen.

The treatment for preterm labor includes bed rest and hydration. Preterm labor before the 20th week is indicative of a nonviable fetus. Bed rest and hydration are nonpharmacological interventions for treating preterm labor. Early labor pains before 20th week are indicative of a nonviable fetus and should generally be uninterrupted. A preterm labor may result in neonatal death, so pregnancy should be maintained to prevent neonatal death. The substantial uterine contractions before the 37th week of gestation leads to preterm labor. Preterm labor has significant poorer cognitive effects on premature babies.

Which statements relate to preterm labor? Select all that apply. A premature baby has good cognitive development. The treatment for preterm labor includes bed rest and hydration. Preterm labor before the 20th week is indicative of a nonviable fetus. It is not desirable to stop the delivery in the case of preterm labor. Preterm labor refers to uterine contractions progressing to delivery before the 27th week of pregnancy.

Specific immune globulin Specific immune globulins contain a high concentration of antibodies directed at specific antigens. Toxoid vaccines contain a bacterial toxin that has been changed to a nontoxic form. Killed vaccines contain killed microbes or isolated microbes. Live attenuated vaccines are composed of live microbes that have been weakened or rendered completely avirulent.

Which type of immune preparation, made from donated blood, contains antibodies that provide passive immunity? Toxoid Killed vaccine Live attenuated vaccine Specific immune globulin

Papillomavirus vaccine (Gardasil) Human papillomavirus virus (HPV) is a common cause of genital warts and cervical cancer. The HPV vaccine (Gardasil, Cervarix) is the first and only vaccine known to prevent cancer.

Which vaccination is marketed and recommended in the prevention of a virus that is known to cause cervical cancer? Papillomavirus vaccine (Gardasil) Herpes zoster vaccine (Zostavax) Pneumococcal vaccine (Prevnar 13) Hepatitis B virus vaccine (Recombivax HB)

Hib conjugate vaccine H. influenzae type b (Hib) (HibTITER, ActHIB, Liquid PedvaxHIB) vaccine is a noninfectious, bacteria-derived vaccine. Before this vaccine was developed, infections caused by Hib were the leading cause of bacterial meningitis in children 3 months to 5 years of age.

Which vaccination was developed to prevent bacterial meningitis caused by Haemophilus influenzae? Hepatitis B vaccine Hib conjugate vaccine Gardasil Prevnar

"The medications that I take will help prevent my body from attacking my new kidney." Immunosuppressant therapy is initiated to inhibit optimal immune response. This is necessary in the case of transplantation, because the normal immune response would cause the body to recognize the new tissue as foreign and attack it. The body will identify the new kidney as foreign and will not treat it as the original kidney. While patients with transplants must be careful about exposure to others, especially those who are or might be ill, and practice adequate and consistent infection control techniques, they don't have to avoid people or social interaction. The new kidney brings foreign cells regardless of relationship between donor and recipient.

While caring for a patient preparing for a kidney transplant, the nurse knows that the patient understands teaching on immunosuppression when she states which of the following? "My body will treat the new kidney like my original kidney." "I will have to make sure that I avoid being around people." "The medications that I take will help prevent my body from attacking my new kidney." "My body will only have a problem with my new kidney if the donor is not directly related to me."

Oral contraceptive pills Oral contraceptive pills prevent ovulation, are easy to stop, and are 99% effective in pregnancy prevention. Intrauterine devices, coitus interruptus, and natural family planning will not prevent ovulation; they should not be recommended for this college student who desires a reliable method of birth control that can be easily discontinued.

You are working with a college student who is planning to become sexually active. She is requesting a reliable method of birth control that could be easily discontinued if necessary. Which option should be given the strongest recommendation? Intrauterine device (IUD) Coitus interruptus Natural family planning Oral contraceptive pills

immune response

a cascade of biochemical events that occurs in response to entry of an antigen (foreign substance) into the body; key processes of the immune response include phagocytosis of foreign microorganisms and synthesis of antibodies that react with specific antigens to inactivate them. immune response centers on the blood but may also involve the lymphatic system and the reticuloendothelial system

germ cell

a cell containing half the number of chromosomes of a somatic cell and able to unite with one from the opposite sex to form a new individual; a gamete.

sexuality

a central aspect of being human throughout life encompasses sex, gender identities and roles, sexual orientation, eroticism, pleasure, intimacy and reproduction

antihypertensives

a class of medication that can be beneficial by reducing BP and minimizing further bleeding and vascular damage in patients with a hemorrhagic stroke; contraindicated in other types of stroke; labetolol is a commonly used agent

analgesics

a class of medication that controls pain for patient comfort as well as being used to avoid comprising sufficient oxygen delivery to ischemic cells; fentanyl and morphine are commonly used agents.

antipyretics

a class of medication that counteracts the increased metabolic demand of a fever post-stroke and severe head injury; acetaminophen is a commonly used agent

antiparkinsonian agents

a class of medication that either replace or delay the breakdown of dopamine; Levodopa is a commonly used agent that is converted to dopamine in the CNS, where it serves as a neurotransmitter

cholinesterase inhibitors

a class of medication that is prescribed to dementia patients in order to increase the availability of acetylcholine, improving cortical function

glucocorticoids

a class of medication that may be effective in reducing cerebral edema related to tumors, abscesses, and CNS infections; contraindicated in some cases and are associated with a poor outcome in treating moderate to severe head injury; dexsmethasone is a commonly used agent

antiepileptics

a class of medication that stabilizes nerve cell membranes and prevents the distribution of epileptic discharges, which can exacerbate or cause increase ICP and increase oxygen demand; phenytoin and valproic acid are commonly used agents

Cushing's triad

a clinical set of symptoms consisting of: Irregular, decreased respirations (caused by impaired brainstem function) Bradycardia Systolic hypertension (widening pulse pressure)

influenza virus (flu)

a contagious respiratory virus that infects the nose, throat, and sometimes the lungs. It can cause mild to severe illness, and at times can lead to death. The best way to prevent this virus is by getting an annual vaccine.

shunt

a hole or a small passage which moves, or allows movement of CSF from an area of the brain to an extracranial location

exudate

a mass of WBCs and serous fluid that has seeped out of blood vessels or an organ

immunity

a physiological process that provides an individual with protection or defense from disease

pessaries

a prosthetic device inserted into the vagina to reduce the protrusion of pelvic structures into the vagina

booster shot

a repeat dose of an antigen, such as a vaccine or toxoid, which is usually administered in an amount smaller than that used in the original immunization. it is given to maintain the immune response of a previously immunized patient at, or return the response to, a clincially effective level.

Rest, Ice, Compression, Elevation (RICE)

a series of activities directed at minimizing the swelling (and therefore the pain) associated with a sprain or strain; most beneficial during the first 24-48 hours after injury

TORCH antibody panel

a series of blood tests that searches for the presence of antibodies to toxoplasmosis, rubella, cytomegalovirus (CMV), and herpes simplex

sexual health

a state of well-being in relation to sexuality across the lifespan that involves physical, emotional, mental, social and spiritual dimensions

decompressive craniectomy

a surgical intervention that removes the rigid confines of the skull allowing for expansion of the cranial contents and lowering ICP

male condom

a type of contraceptive that prevents pregnancy and STIs`

antivenin

an antiserum against a venom used to treat humans or other animals that have been envenomed (e.g., by snakebite, spider bite, or scorpion sting); a type of passive immunizing drug

primary immunodeficiency

an innate suppressed immune response

antibodies

immunoglobin molecules that have an antigen-specific amino acid sequence and are synthesized by the humoral immune system; levels may decline with age

chronic inflammation

inflammation that continues for weeks to years after initial injury

adenopathy

large or swollen lymph nodes; a minor adverse effect of vaccine administration

Rh₀(D) immunoglobulin (RhoGAM, WinRho)

passive immunity used to suppress the formation of anti-Rh₀(D) antibodies in an Rh₀(D)-negative person exposed to Rh-positive blood, which can be fatal. The most common use of this product is in cases of maternal-fetal Rh incompatibility (postpartum).

antitoxin

purified antiserum obtained from animals (usually horses) by injection of a toxin or toxoid so that antibodies to the toxin can be collected from the animals and used to provide artificial passive immunity to humans exposed to a given toxin (e.g., tetanus, immunoglobin); a type of passive immunizing drug

herd immunity

resistance to a disease on the part of an entire community or population because a large proportion of its members are immune to the disease (≈95%)

immunization

the induction of immunity by administration of a vaccine or toxoid (active) or antiserum (passive)

infection

the invasion and multiplication of microorganisms in body tissues, which may be unapparent or the result of local cellular injury caused by competitive metabolism, toxins, intracellular replication, or antigen-antibody response.

Natural active immunization

type of immunization in which the person acquires immunity by surviving the disease itself and producing antibodies to the disease-causing organism

Artificial passive immunization

type of immunization which involves the administration of serum or concentrated immunoglobulins; the inoculated person is given the substance needed to fight off the invading microorganism; this type of immunization bypasses the host's immune system.

measles, mumps, and rubella virus vaccine (M-M-R II)

type of vaccine: -active immunity -contraindicated in persons with a history of anaphylactic or anaphylactoid ractions, egg allergy, or pregnancy -pregnancy should be avoided for 3 months after vaccination

rabies virus vaccine (Imovax, Rab-Avert)

type of vaccine: -active immunity -current recommendedations call for a total of five IM injections on days 0, 3, 7, 14, and 28 following a concerning animal bite -preexposure vaccine is recommended for person at high risk exposure to the virus (e.g., veterinarians) and consists of three injections and day 0, 7, and sometime between 21 and 28. booster shots are recommended approx. every 2 to 5 years. -due to the life-threatening nature of the infection, treatment may still be required despite contraindications

tetanus toxoid (Daptacel)

type of vaccine: -active immunity -given after 6 weeks of age -given with diphtheria and pertussis (whooping cough) -booster required approx. every 10 years -contraindicated in pregnant women during the first trimester

varicella virus vaccine (Varivax)

type of vaccine: -active immunity -given at 12 to 15 months of age, and a second dose given at 4-6 years of age -contraindicated in immunodeficient patients and patients who have received high doses of systemic steroids in the previous month -used to prevent chickenpox

poliovirus vaccine, inactivated (IPOL)

type of vaccine: -active immunity -live vaccine is no longer routine in the United States -no contraindictors

human papillomavirus (HPV) vaccine (Gardasil, Cervarix)

type of vaccine: -active immunity -prevents a common cause of genital warts and cervical cancer -recommended for girls 11-26 and boys 11-21, men who have had sex with other men, and men with compromised immune systems (e.g., HIV) -three injections: the first dose, and then two more doses 2 months later and 6 months later -contraindicated in persons with yeast allergies, and pregnant women -pain is common on injection

pneumococcal vaccine (Pneumovax 23, PVC13, Prevnar 13)

type of vaccine: -active immunity -prevents pneumonia, meningitis, and sepsis -recommended for pediatric patients at high risk for pneumonia as a result of serious chronic illnesses, especially those who are immunocompromised -recommended for smokers 19-64 -contraindicated in persons that are immunosuppressed as a result of drug therapy (e.g., chemotherapy)

rabies immunoglobulin (BayRab, Imogam Rabies-HT)

used concurrently with rabies virus vaccine to provide passive immunity following suspected exposure to the rabies virus

hepatitis B immunoglobulin (BayHep B, Nabi-HB)

used to provide passive immunity to persons exposed to hepatitis B infection


Kaugnay na mga set ng pag-aaral

ID Assessment: Short-Answer Questions

View Set

NURS 309 Acute Type 1 Diabetes-1

View Set

Week 6 Appendicular (Mastering A&P)

View Set

Technology for Success Module 6 Quiz (Security and Safety)

View Set